Prove sequence converges to sup

  • Thread starter CAF123
  • Start date
  • Tags
    Sequence
In summary: Good work!In summary, we have proven that given a nonempty bounded set E contained in ##\mathbb{R}## with ##\sup E## not in E, there exists a strictly increasing sequence ##\left\{x_n\right\}## that converges to ##\sup E## and has all its elements in E. This is accomplished by using the Approximation property for suprema and the nested intervals technique.
  • #1
CAF123
Gold Member
2,948
88

Homework Statement


Suppose that E is contained in ##\mathbb{R}## is a nonempty bounded set and that ##\sup E## is not in E. Prove that there exists a strictly increasing sequence ##\left\{x_n\right\}## that converges to ##\sup E## such that ##x_n \in E## for all n in ##\mathbb{N}##.

Homework Equations


More like Relevant theorems:
Bolzano-Weirstrass,
Completeness,
Monotone convergence.

The Attempt at a Solution



E is contained in R and nonempty so by Completeness, sup E exists. E is bounded so -a < e < a for all e in E and let supE = a which is not in E. I suppose this might be the essence of the proof, but how exactly do we know such a sequence in E exists? If there existed a function ##f: \mathbb{N} \rightarrow \mathbb{R}## with all the ##x_n## in E, then provided I can show such a sequence is strictly increasing (and that it exists) then the rest of the problem becomes trivial. I thought about using nested intervals but I didn't get very far. Any ideas?

Many thanks.
 
Physics news on Phys.org
  • #2
Here's an outline/suggestion on how to proceed: let [itex]sup(E)=s_E[/itex].
1) Can you show that given any [itex]\epsilon>0[/itex], there exists an [itex]x\in E[/itex] such that [itex]s_E-\epsilon<x[/itex]?
2) Consider the sequence [itex]\epsilon_n=\frac{1}{n}[/itex]; apply this sequence of epsilons to part 1 to show that you can generate a sequence in [itex]E[/itex], call it [itex]x_n[/itex].
3) Show that this sequence converges to [itex]sup(E)=s_E[/itex].

Hint for part 1): this is a good exercise in proof by contradiction which teaches you how to negate quantifiers; "for all"'s become "there exists" and vice-versa, and statements get negated.
 
  • #3
christoff said:
Here's an outline/suggestion on how to proceed: let [itex]sup(E)=s_E[/itex].
1) Can you show that given any [itex]\epsilon>0[/itex], there exists an [itex]x\in E[/itex] such that [itex]s_E-\epsilon<x[/itex]?
Yes, this is the Approximation property for suprema

2) Consider the sequence [itex]\epsilon_n=\frac{1}{n}[/itex]; apply this sequence of epsilons to part 1 to show that you can generate a sequence in [itex]E[/itex], call it [itex]x_n[/itex].
I can make the following statements: supE - 1 < x for some x, supE - 1/2 < x for some x, etc.. So supE-1, supE-1/2,...supE-1/n.. would make a sequence and indeed it would be strictly increasing. My only worry is that how can I be sure that the elements would be in E?
 
  • #4
CAF123 said:
Yes, this is the Approximation property for supremaI can make the following statements: supE - 1 < x for some x, supE - 1/2 < x for some x, etc.. So supE-1, supE-1/2,...supE-1/n.. would make a sequence and indeed it would be strictly increasing. My only worry is that how can I be sure that the elements would be in E?

The numbers [itex]Sup(E)-1, Sup(E)-1/2, Sup(E)-1/3,...[/itex] aren't the numbers you're interested in; it's the x's that bound them that you want to look at.

Let's start at n=1. You said that there exists an [itex]x\in E[/itex] such that [itex]Sup(E)-1<x[/itex]. Let's call this x we found [itex]x_1[/itex].
Now, n=2. We know that there exists an [itex]x\in S[/itex] such that [itex]Sup(E)-1/2<x[/itex]. Let's call this second x we've found [itex]x_2[/itex].
Then we could do the same for n=3, n=4, etc.

The important part thing is we can do this for ANY n; so in particular, there exists a sequence [itex]x_n\in E[/itex] with the property that [itex]Sup(E)-1/n<x_n[/itex]. Can you show that this sequence converges? Hint below if you can't get it.

You also know that, because [itex]Sup(E)[/itex] is the supremum, that each [itex]x_n[/itex] is bounded by [itex]Sup(E)[/itex]. So you have [itex]Sup(E)-1/n<x_n\leq Sup(E)[/itex].
 
Last edited:
  • #5
christoff said:
The numbers [itex]Sup(E)-1, Sup(E)-1/2, Sup(E)-1/3,...[/itex] aren't the numbers you're interested in; it's the x's that bound them that you want to look at.

Let's start at n=1. You said that there exists an [itex]x\in E[/itex] such that [itex]Sup(E)-1<x[/itex]. Let's call this x we found [itex]x_1[/itex].
Now, n=2. We know that there exists an [itex]x\in S[/itex] such that [itex]Sup(E)-1/2<x[/itex]. Let's call this second x we've found [itex]x_2[/itex].
Then we could do the same for n=3, n=4, etc.

The important part thing is we can do this for ANY n; so in particular, there exists a sequence [itex]x_n\in E[/itex] with the property that [itex]Sup(E)-1/n<x_n[/itex]. Can you show that this sequence converges? Hint below if you can't get it.

You also know that, because [itex]Sup(E)[/itex] is the supremum, that each [itex]x_n[/itex] is bounded by [itex]Sup(E)[/itex]. So you have [itex]Sup(E)-1/n<x_n\leq Sup(E)[/itex].

Ok so the last statement implies $$\lim_{n \rightarrow \infty} \sup E - \lim_{n \rightarrow \infty} \frac{1}{n} \leq \lim_{n \rightarrow \infty} x_n$$ From this ##x_n## must converge to ##\sup E##. It caanot be greater because the ##x_n## are bounded above by ##\sup E##. Is this all I need to do? I haven't checked your hint yet, I prefer to have feedback first.
 
  • #6
CAF123 said:
Ok so the last statement implies $$\lim_{n \rightarrow \infty} \sup E - \lim_{n \rightarrow \infty} \frac{1}{n} \leq \lim_{n \rightarrow \infty} x_n$$ From this ##x_n## must converge to ##\sup E##. It caanot be greater because the ##x_n## are bounded above by ##\sup E##. Is this all I need to do? I haven't checked your hint yet, I prefer to have feedback first.

Yes, that is basically the idea. Since the sequence [itex]sup(E)-1/n[/itex] converges to [itex]sup(E)[/itex], and [itex]x_n[/itex] is bounded above by the [itex]sup(R)[/itex] and below by this [itex]sup(E)-1/n[/itex], it must converge to [itex]sup(E)[/itex].
 

1. What is a sequence?

A sequence is a list of numbers that are ordered according to a specific rule or pattern. Each number in the sequence is called a term.

2. What does it mean for a sequence to converge?

A sequence converges if its terms get closer and closer to a single number as the sequence goes on. This single number is called the limit of the sequence.

3. What is the supremum (sup) of a sequence?

The supremum, or sup, of a sequence is the smallest number that is greater than or equal to all the terms in the sequence. It can also be thought of as the limit of the sequence if it converges.

4. How do you prove that a sequence converges to its supremum?

To prove that a sequence converges to its supremum, you must show that the terms of the sequence get closer and closer to the supremum as the sequence goes on. This can be done by using the definition of convergence and the properties of the supremum.

5. What are the applications of proving a sequence converges to its supremum?

Proving that a sequence converges to its supremum is important in various mathematical and scientific fields. It can be used to prove the existence of solutions to equations, analyze the behavior of functions, and make predictions in real-world situations involving changing quantities.

Similar threads

  • Calculus and Beyond Homework Help
Replies
4
Views
881
  • Calculus and Beyond Homework Help
Replies
13
Views
964
  • Calculus and Beyond Homework Help
Replies
11
Views
2K
  • Calculus and Beyond Homework Help
Replies
8
Views
2K
  • Calculus and Beyond Homework Help
Replies
5
Views
1K
  • Calculus and Beyond Homework Help
Replies
14
Views
2K
  • Calculus and Beyond Homework Help
Replies
4
Views
305
  • Calculus and Beyond Homework Help
Replies
7
Views
706
  • Calculus and Beyond Homework Help
Replies
1
Views
1K
  • Calculus and Beyond Homework Help
Replies
3
Views
1K
Back
Top